What is the range of the following data set?


5, 6, 7, 3, 4, 5, 6, 8, 7



A) 2



B) 5



C) 3



D) 0

Answers

Answer 1
Highest number - lowest number = 8-3 = 5

I hope that helps and best of wishes !!

Related Questions


Doug needs to order 25,000 g of a
supplement. At the store,
supplements come in 5 kg packages
that cost $80 each. What will Doug's
order cost?

Answers

Answer:

$2000

Step-by-step explanation:

1,000 g would be = 1 Kg so it would be 25 Kg

And 25 X 80 = 2000

Answer:

$400

Step-by-step explanation:

25,000 grams is equal to 25 kg, so if 5 kg equals $80, the total 25 kg divided by 5 equals five. With this you multiply; 5 × 80 kg = 400. So $400 is the answer.

HELPPPPPPPPPPPPPPPPPPPPPPPPPPPPPPPPPPPPPPPPPP

Answers

Answer:

Option B is the correct answer

Step-by-step explanation:

A line segment is a line drawn with two end points.

1.) Option A is a point

2.) Option B is a line segment

3.) Option C is a ray

4.) Option D is a line

Hope this helps!

Answer Please.......​

Answers

Answer:

7.1

Step-by-step explanation:

Use the trig ratio tan to find b

4.5 x tan (23) = b = 7.14668887526

7.1

Answer:

b ≈ 1.9

Step-by-step explanation:

Using the tangent ratio in the right triangle

tan23° = [tex]\frac{opposite}{adjacent}[/tex] = [tex]\frac{b}{4.5}[/tex] ( multiply both sides by 4.5 )

4.5 × tan23° = b , then

b ≈ 1.9 ( to the nearest tenth )

Identify the graph of the equation (x+1)2 + (y - 3)2 = 9.

Answers

Answer:

A circle centred at (-1,3) and radius 3.

Step-by-step explanation:

There are
15
apples in a basket.
6
of these apples are green. The rest of them are red.
(a) What is the ratio of red apples to all apples in the basket?
(b) What is the ratio of green apple to red apples?

Answers

Answer:

 (a) 9:15

  (b) 6:9

Step-by-step explanation:

Total apples: 15

6 : green

9 are red

9:15

6:9

Answer:

(a) 3 : 5

(b) 2 : 3

Step-by-step explanation:

green apples = 6 total apples = 15 red apples = 15 - 6

= 9 apples

(a) red apples : all apples

= 9 : 15

for simplified form dividing numerator and denominator by 3

= 3 : 5

(b) green apples : red apples

= 6 : 9

for simplified form dividing numerator and denominator by 3

= 2 : 3

In a certain class, percentage of students who increased their average grade during spring term compared to that of the fall term was recorded between 6.9% and 7.1%. What could be the least possible number of students in that class?

Answers

Answer:

No, you have to consider that if 1 student in 14/15 cannot represent a percentage between .69 and .71, how could 2 students do the job? You have to have an integer number of students.

2/18 = 1/14 = 7.14%

2/29 = 6.89%

Still out of bounds

What about 3 students?

3/42 = 1/14 = 7.14%

3/43 = 6.97%

3/44 = 6.81%

So, it looks like 43 is the minimum number of students.

i need help with my math please help me understand. Itook a picture of it.

Answers

Answer:

1/3 x 1/3 x 1/3(volume of small cube) = 1/27

2/3 x 7/3 x 4/3(volume of figure shown) = 2 2/27

Now we divide to see how many cubes can fit.:

2 2/27(make improper fraction) divided by 1/27 = 56 small little cubes can fit.

[tex]4x^{2} -4x+1=0[/tex]

Answers

Answer:

x=1/2

Step-by-step explanation:

See image below:)

Nelda baked two kinds of pasta in pans. Each pan was the same size. She sliced one pan of pasta into 4 equal pieces. She sliced the other pan into 6 equal pieces. How can the pans of pasta now be sliced so that both pans have the​ same-sized pieces? If Nelda has served 8 pieces from 1 pan so​ far, what fraction of 1 pan has she​ served?

Answers

Answer:

6 equal parts;  8 equal parts

Both those will go into 24.

Step-by-step explanation:  Each of six slices of the first pasta has the area 1%2F6 of the entire pan.

Each of eight slices of the second pasta has the area 1%2F8 of the original pan.

Nelda will cut each 1%2F6 in 4 equal pieces to get 1%2F24.

She also will cut each 1%2F8 in 3 equal pieces to get 1%2F24.

solve the equation by completing the square. Round to the nearest hundredth if necessary.


x^2+2x=8


A. X= 2, 4

B. X= -2, 4

C. X= 2, -4

D. X= -2, -4

Answers

This answer would be A hope this helps

_____ + 15 − 15 = 21
help mee

Answers

Answer:

x = 21

Step-by-step explanation:

x + 15 - 15 = 21

+ 15 and - 15 cancel out

So x = 21

Let's solve the longer way

x + 15 - 15 = 21

x + 15 = 21 + 15 .      (adding 15 to both sides)

x = 21                       (Subtracting 15 from both sides)

If my answer is incorrect, pls correct me!

If you like my answer and explanation, mark me as brainliest!

-Chetan K

Answer:

The answer that goes into the blank is 21

Step-by-step explanation:

15-15 is 0, so 21+15= 36-15=21

What is the range of the function represented by the graph?
2
2
-2
-2
O {0, 1, 2, 3}
O {1, 2, 3, 4)
O 1sys4
O OSXS3

Answers

Answer:

Range: { 1,2,3,4}

Step-by-step explanation:

The range is the values that y can take

These are discrete points, so y is discrete values

Range: { 1,2,3,4}

please help im not sure if my answer is correct last test of the year really important answer for brainliest

Answers

Answer:

That's correct

Well done!!

That is right

Good job I hope you did good on the rest of your test :)

A ball is thrown up in the air, and it's height (in feet) as a function of time (in seconds) can be written as h(t) = -16t2 + 32t + 6. Describe in detail (using formulas and/or specific steps needed) TWO ways to find the maximum height reached by the ball.

Answers

Answer:

1) Using properties of the quadratic equation:

Here the height equation is:

h(t) = -16t^2 + 32t + 6

We can see that the leading coefficient is negative, this means that the arms of the graph will open downwards.

Then, the vertex of the quadratic equation will be the maximum.

Remember that for a general quadratic equation:

a*x^2 + b*x + c = y

the x-value of the vertex is:

x = -b/2a

Then in our case, the vertex is at:

t = -32/(2*-16) = 1

The maximum height will be the height equation evaluated in this time:

h(1) = -16*1^2 + 32*1 + 6 = 22

The maximum height is 22ft

2) Second method, using physics.

We know that an object reaches its maximum height when the velocity is equal to zero (the velocity equal to zero means that, at this point, the object stops going upwards).

If the height equation is:

h(t) = -16*t^2 + 32*t + 6

the velocity equation is the first derivation of h(t)

Remember that for a function f(x) = a*x^n

we have that:

df(x)/dx = n*a*x^(n-1)

Then:

v(t) = dh(t)/dt = 2*(-16)*t + 32 + 0

v(t) = -32*t + 32

Now we need to find the value of t such that the velocity is equal to zero:

v(t) = 0 = -32*t + 32

       32*t = 32

            t = 32/32 = 1

So the maximum height is at t = 1

(same as before)

Now we just need to evaluate the height equation in t = 1:

h(1) = -16*1^2 + 32*1 + 6 = 22

The maximum height is 22ft

factor the polynomial below completely.

z^2 + 9z + 20

Answers

Answer:

Step-by-step explanation:

z^2+9z+20

=z^2+(5+4)z+20

=z^2+5z+4z+20

=z(z+5)+4(z+5)

=(z+5)(z+4)

Help please me please please need explanation

Answers

9514 1404 393

Answer:

  a) see attached

  b) 1/3

Step-by-step explanation:

Your grid doesn't come with any coordinate axes, so you will need to draw those and label them. You will also need to determine the number of grid squares for each unit. (I suggest 2 grid squares per unit to give you a little more distance between the points you plot. The attachment shows 5 small squares per unit. You may not have this much space on your graph.)

Plot the points. Once you do this, you can see that their horizontal difference is 3 units and their vertical difference is 1 unit. The right point is above the left point, so the slope is positive:

  m = (rise)/(run) = 1/3 . . . slope of the line

Math question please show work due today last day to submit Assignments or I fail please help

Answers

Answer:

The polygons are similar.

Step-by-step explanation:

By definition, the corresponding sides and angles of similar polygons must be in some constant proportion.

Therefore, we can find the ratio between corresponding sides to see if that ratio is maintained.

The side marked 3 cm in the smaller figure corresponds with the side marked 3.75 cm in the larger figure. This proportion is [tex]\frac{3}{3.75}[/tex].

The side marked 5 in the smaller figure corresponds with the side marked 6.25 cm in the larger figure. This proportion is [tex]\frac{5}{6.25}[/tex].

If these polygons are similar, these proportions must be equal.

[tex]\frac{3}{3.75}=\frac{5}{6.25},\\0.8=0.8\:\checkmark[/tex]

Checking, we see that these proportions are equal, and therefore the polygons are similar.

James is using cement to make a new sidewalk and new steps.
Part A
He will make a new rectangular sidewalk that is 8 feet long, 4 feet wide, and 0.25 foot thick.
What is the volume, in cubic feet, of the cement used to make the sidewalk? Show or explain all of the steps you used to determine your answer.

Part B
This diagram shows James’s design for the steps. The height of each step will be 6 inches. All angles in the diagram are right angles.
I need it now please!

Answers

The volume, of the cement used to make the sidewalk is 8 ft³

What is volume?

Volume is defined as the space occupied within the boundaries of an object in three-dimensional space. It is also known as the capacity of the object.

Given that, James is using cement to make a new sidewalk and new steps,

He will make a new rectangular sidewalk that is 8 feet long, 4 feet wide, and 0.25 foot thick.

Volume = length x width x height

Volume = 8 x 4 x 0.25 = 8 ft³

Hence, the volume, of the cement used to make the sidewalk is 8 ft³

Learn more about volumes, click;

https://brainly.com/question/1578538

#SPJ1

For the following right triangle, find the side length x. Round your answer to the nearest hundredth. X, 18, and 10

Answers

Answer:

152

Step-by-step explanation:

All segments in a triangle add to 180. 1

8+10+x=180

x=152

The length x using Pythagoras theorem is 15 unit.

What is Pythagoras theorem?

According to the Pythagorean theorem, the square of the hypotenuse of a right triangle equals the sum of the squares of the other two sides.

Given:

Hypotenuse = 18 unit

Perpendicular = 10 unit

using Pythagoras theorem

H² = P² + B²

18² = 10² + B²

324 = 100 + B²

324 - 100 = B²

B² = 224

B= 14.966 unit

B= 15 unit

Thus, the measure for x is 15 unit.

Learn more about Pythagoras theorem here:

https://brainly.com/question/343682

#SPJ7

raja bought 500 hens among them 75 were died by cold . he sold the remaining hens at the rate of rs 120 per hen and made a profit of 2% find the total CP of hens?plz guys help me tomorrow is my exam and it is fix question. so give me answer as soon as possible​

Answers

CP hens remianing = 5000

Match each expression with its sum or difference

Answers

Answer:

1-4=-3

-1-4=-5

-1+4=3

1-|-4|=5

Answer:

[tex]1-4=-3\\-1-4=-5\\-1+4=3\\1-(-4)=5[/tex]

Step-by-step explanation:

The hardest part about negative numbers in addition is remembering when to add, rather than subtract. In the last equation, we add the numbers together, because the negative sign cancels out with subtraction. All of the other equations we can follow exactly what it says to do, because there are no minus signs right next to each other.

Tao uses elimination to solve the following system of linear equations.

Answers

Given:

The system of equation is:

[tex]-2x+4y=16[/tex]

[tex]2x+2y=8[/tex]

To find:

The solution of the given system of equations.

Solution:

We have,

[tex]-2x+4y=16[/tex]              ...(i)

[tex]2x+2y=8[/tex]                ...(ii)

Add (i) and (ii) to eliminate the variable x.

[tex]6y=24[/tex]

[tex]y=\dfrac{24}{6}[/tex]

[tex]y=4[/tex]

Putting [tex]y=4[/tex] in (ii), we get

[tex]2x+2(4)=8[/tex]

[tex]2x+8=8[/tex]

[tex]2x=8-8[/tex]

[tex]2x=0[/tex]

Divide both sides by 2.

[tex]x=0[/tex]

Therefore, the solution of given system of equations is (0,4). Hence, the correct option is A.

Answer:

(0,4) or A

Step-by-step explanation:

Edge 2021

The sum of two rational numbers will always be
O an irrational number.
an integer
a rational number
a whole number.

Answers

Step-by-step explanation:

The sum of two rational numbers will always be a rational number.

What is the value of the product (3 - 2i)(3 + 2i)

Answers

Answer:

13

Step-by-step explanation:

(3 - 2i)(3 + 2i)

FOIL

first  3*3 = 9

outer 3*2i = 6i

inner -2i*3 = -6i

last  -2i * 2i = -4i^2 = -4(-1) =4

Add together

9+6i-6i+4 = 13

30 pts 5 stars thanks and brainliest please- this question is really hard...

Answers

Answer:

From least to greatest

C

B

A

Step-by-step explanation:

Function A is the only one with a positive y-intercept

so it's the greatest

Function B has a y-intercept of -1 as read of the graph

it is the middle one

Function C has a y-intercept of -9/4 as given in point (0, -9/4)

this is the least

-----------------

From least to greatest

C

B

A

−n+(−4)−(−4n)+6 find equivalent expressions PLZ

Answers

Answer:

3n+2

Step-by-step explanation:

−n+(−4)−(−4n)+6

Combine like terms

−n−(−4n)+6+(−4)

A negative negative is like adding

−n+(4n)+6+(-4)

3n+2

The answer to your question is 3n+2

What is the approximate value of sin C?

Answers

Answer: “D. 0.36”

Explanation:
Sin is opposite over hypotenuse!
5/13.93 = 0.358938
Rounded to the nearest hundredth is 0.36 :)

3
33%
A fair dice has six sides, numbered 1 to 6
After it is rolled, five of the numbers can be seen.
ws
a) Write down the probability that one of these five numbers is 3
(1)
b) Work out the least possible sum of the five numbers.
(2)

Answers

Answer:

a) 5/6 probability that one of the five numbers is 3

b) the least possible sum of the five numbers will be 15

as the least will come from the lowest 5 numbers

you have at most $20.25 to spend at a store. Shirts cost $11.50 each, and pants cost $16 each. Let s be numbers of shirts and p be number of pants. Write an inequality that represents the numbers of shirts and pants you can afford. DO NOT INCLUDE THE DOLLAR SIGN IN YOU INEQUALITY. An inequality is______

Answers

is this an algebra class? if so it just

11.50x + 16y = 20.25

1⁄2 × 1⁄5 = ....whats this answer

Answers

Answer:

0.1

Step-by-step explanation:

Answer:

√2 = 1.41

√5 = 2.23

SO IF YOU MULTIPLY SEE

3.1622776601683793319988935444327

Step-by-step explanation:

Other Questions
Suppose the current level of output is 5000. If the elasticities of output with respect to capital and labor are 0.3 and 0.7, respectively, a 10% increase in capital combined with a 5% increase in labor and a 5% increase in productivity would increase the current level of output to :_______ You are considering buying a share of stock in a firm that has the following two possible payoffs with the corresponding probability of occurring. The stock has a purchase price of $15.00. You forecast that there is a 40% chance that the stock will sell for $30.00 at the end of one year. The alternative expectation is that there is a 60% chance that the stock will sell for $10.00 at the end of one year. What is the expected percentage oneyear return on this stock, and what is the return standard deviation? I really need help on this question and nooo links the question is in the screenshotbiomagnificationeutrophicationglobal warmingbiogeochemical cycles PLEASE HELP ON ANY WILL GIVE BRIANLIEST Pangaea was a ... that formed near the end of the Paleozoic era. a) Controls one-directional transmission of impulses Find the volume of a right circular cone that has a height of 3.9 ft and a base with a radius of 14.7 ft. Which graph represents the given inequality? y > 4x + 2 An empty paper cup is the same temperature as the air in the room. A student fills the cup with cold water. Which of the followingdescribes how thermal energy is transferred?A. Thermal energy is transferred from the cold water to the cup until they are at the same temperature.B. Thermal energy is transferred from the cup to the cold water until they are at the same temperature.C. Thermal energy is transferred from the cup to the cold water until the cup has no more thermal energyD. Thermal energy is not transferred between the cup and the cold water. What were some questions related to policies and economics people making after WW II? Sundayf depth25. Based on what you know about the elements of art, and what you see in Sunday Afternoon onthe Island of La Grande Jatte, which element do you believe Seurat used most successfully in hiswork? Why? Explain. Exercise : Randomizer In this exercise, we are going to create a static class Randomizer that will allow users to get random integer values from the method nextInt() and nextInt(int min, int max). Remember that we can get random integers using the formula int randInteger = (int)(Math.random() * (range + 1) + startingNum). nextInt() should return a random value from 1 - 10, and nextInt(int min, int max) should return a random value from min to max. For instance, if min is 3 and max is 12, then the range of numbers should be from 3 - 12, including 3 and 12. What is the purpose of a conclusion?O A. To predict how something will turn outB. To explain why the data support or reject the hypothesisC. To make an educated guess about a particular questionD. To collect or analyze the experimental data Any help 70 points Which of the following is NOT a factor that affects Elasticity of Demand 1. availability of substitutes (how many items you could buy instead) 2. necessity vs. luxury item / service (how much you need the item3 time horizon (how long you have to buy) it 4. taxes (how much the 5. government adds to the price Why is the Jamaican Parliament described as Bicameral Parliament? Help me...! !!!!!!!! Is North Korea planning a Genocide on the U.S?They spread false information and feed the people living there wrong information in schools. They tell them lies about the U.S., and encourage them to hate us. Has anyone found out, why, they are doing this? Why, then, do you think the Rose Center exhibit caused such a stir? A eukaryotic mutation upstream of a particular gene has been identified that changes the sequence of the TATA box to GATA. How would you predict that this mutation would affect the transcription of this gene Show that p-1 is a factor of p10 -1.